Download as pdf or txt
Download as pdf or txt
You are on page 1of 25

MA 1014-Mathematics Riemann Integration Dr Miuran Dencil

0.1 Introduction
Let f : [a, b] → R be a bounded (not necessarily continuous) function on a compact (closed, bounded) interval.
We will define what it means for f to be Riemann integrable on [a, b] and, in that case, we define its Riemann
integral
Z b
f.
a

The integral of f on [a, b] is a real number whose geometrical interpretation is the signed area under the graph
y = f (x) for a ≤ x ≤ b. This number is also called the definite integral of f . By integrating f over an interval
[a, x] with varying right end-point, we get a function of x, called the indefinite integral of f .

The most important result about integration is the fundamental theorem of calculus, which states that integra-
tion and differentiation are inverse operations in an appropriately understood sense. Among other things, this
connection enables us to compute many integrals explicitly.

0.2 Definition of Riemann Integration


Two intervals are said to be almost disjoint they are disjoint or intersect only at a common endpoint. For
example, the intervals [1, 3] and [3, 4] are almost disjoint, whereas [0, 4] and [2, 5] are not.

Definition 0.2.1 (Partition) Let I be a nonempty, compact interval (closed and bounded). A partition
of I is a finite collection {I1 , I2 , ..., In } of almost disjoint, nonempty, compact sub intervals whose union is I.

A partition P of [a, b] with subintervals Ik = [xk−1 , xk ] is determined by the set of endpoints of the intervals
a = x0 < x1 < x2 < ... < xn−1 < xn = b.

We will denote a partition P either by its intervals P = {I1 , I2 , ..., In } or by the set of endpoints of the
intervals P = {x0 , x1 , x2 , ..., xn1 , xn }.

We will adopt either notation as convenient but the context should make it clear which one is being used.
There is always one more endpoint than interval.

Example 0.2.1 The set of intervals

{[0, 1/6], [1/6, 1/5], [1/5, 1/4], [1/4, 1/3], [1/3, 1/2], [1/2, 1]}

is a partition of [0,1]. The corresponding set of endpoints are

{0, 1/6, 1/5, 1/4, 1/3, 1/2, 1} .

We denote the length of an interval I = [a, b] by

|I| = b − a.

For the interval I, the sum of the lengths |Ik | = xk − xk−1 of the almost disjoint sub-intervals in a partition
{I1 , I2 , ..., In } is equal to length of the whole interval. This is obvious geometrically; algebraically, it follows
from the telescoping series

n
X n
X
|Ik | = (xk − xk−1 ) (0.1)
k=1 k=1
= xn − xn−1 + xn−1 − xn−2 + ... + x2 − x1 + x1 − x0 (0.2)
= xn − x0 (0.3)
= In . (0.4)

Suppose that f : [a, b] → R is a bounded function on the compact interval I = [a, b] with

M = sup f, m = inf f.
I I

If P = {I1 , I2 , ..., In } is a partition of I, let

1
MA 1014-Mathematics Riemann Integration Dr Miuran Dencil

Mk = sup f = sup {f (x) : x ∈ [xk−1 , xk ]} , mk = inf f = inf {f (x) : x ∈ [xk−1 , xk ]}


Ik Ik

The upper Riemann sum of f with respect to the partition P is defined by


n
X n
X
U (f ; P ) = Mk |Ik | = Mk (xk − xk−1 )
k=1 k=1

Geometrically, U (f ; P ) is the sum of the areas of rectangles based on the intervals Ik that lie above the graph
of f .
The lower Riemann sum of f with respect to the partition P by
n
X n
X
L(f ; P ) = mk |Ik | = mk (xk − xk−1 ).
k=1 k=1

Geometrically, L(f ; P ) is the sum of the areas of rectangles that lie below the graph of f .
For all k,
mk = inf{f (x) : x ∈ [xk−1 , xk ]} ≤ Mk = sup{f (x) : x ∈ [xk−1 , xk ]}.

Lemma 0.2.1 Let f be a bounded function on [a, b] and let P be any partition of [a, b]. Then

L(f, P ) ≤ U (f, P ).

Refinements of partitions
If P and Q are partitions of [a, b], where P ⊆ Q, then Q is called a refinement of P .

Example 0.2.2 Consider the partitions of [0, 1] with endpoints P = 0, 1/2, 1, Q = 0, 1/3, 2/3, 1, R = 0, 1/4, 1/2, 3/4, 1.
Thus, P , Q, and R partition [0, 1] into intervals of equal length 1/2, 1/3, and 1/4, respectively. Then Q is not
a refinement of P but R is a refinement of P.

Note: If P1 and P2 are partitions of [a, b], then Q = P1 ∪ P2 is a refinement of both P1 and P2 .

2
MA 1014-Mathematics Riemann Integration Dr Miuran Dencil

Lemma 0.2.2 If Q is a refinement of P , then


L(f, P ) ≤ L(f, Q) ≤ U (f, Q) ≤ U (f, P )
.
Proof 0.2.1 From the previous lemma, we already know that
L(f ; Q) ≤ U (f ; Q).
We’ll show that L(f ; P ) ≤ L(f ; Q). The proof that U (f ; Q) ≤ U (f, P ) is similar.

Let P = {x0 , x1 , x2 , ..., xn } ⊆ Q. Now ∀k, let Q ∩[xk−1 , xk ] = {xk−1 = xk0 , xk1 , xk2 , ...xkik = xk }. Then
mk = inf{f (x)|x ∈ [xk−1 , xk ]} ≤ mki = inf{f (x)|x ∈ [xk(i−1) , xki ]} f or i = 1, 2, 3, ..ik
Now let,

n
X n
X
L(f ; P ) = mk |Ik | = mk {|Ik1 | + |Ik2 | + |Ik3 | + ... + |Ikik |} (0.5)
k=1 k=1
Xn
≤ {mk1 |Ik1 | + mk2 |Ik2 | + mk3 |Ik3 | + ... + mkik |Ikik |} (0.6)
k=1
ik
n X
X
= mki |Iki | = L(Q; f ). (0.7)
k=1 i=1

Hence
L(f ; P ) ≤ L(f ; Q).
Lemma 0.2.3 If P1 and P2 are any two partitions of [a, b], then
L(f, P1 ) ≤ U (f, P2 )
Proof 0.2.2 Let Q = P1 ∪ P2 be the common refinement of P1 and P2 . By the previous lemmas,
L(f, P1 ) ≤ L(f, Q) ≤ U (f, Q) ≤ U (f, P2 ).
Let P be the collection of all possible partitions of the interval [a, b].
Definition 0.2.2 (Upper and Lower Integrals) The upper integral of f is
U (f ) := inf{U (f, P ) : P ∈ P}.
The lower integral of f is

L(f ) := sup{L(f, P ) : P ∈ P}
A commonly used alternative notation for the upper and lower integrals is
Z b
U (f ) = f
a
Z b
L(f ) = f
a

Lemma 0.2.4 For any bounded function f on [a, b], it is always the case that L(f ) ≤ U (f ).
Definition 0.2.3 A bounded function f : [a, b] → R is Riemann integrable on [a, b] if its upper integral U (f )
and lower integral L(f ) are equal. In that case, the Riemann integral of f on [a, b], denoted by
Z b Z b Z
f (x)dx, f, f
a a [a,b]

An unbounded function is not Riemann integrable. In the following, “integrable” will mean “Riemann integrable,
and “integral” will mean “Riemann integral” unless stated explicitly otherwise.
Riemann integral is defined for f on a closed interval [a, b], f : [a, b] → R.

3
MA 1014-Mathematics Riemann Integration Dr Miuran Dencil

Example 0.2.3
A constant function is Riemann integrable on [a, b]

Solution: Let f (x) = C, ∀x ∈ [a, b] where C ∈ R be a constant.


Clearly f is bounded on [a, b] and inf f = sup f = C.

Let P = {x0 , x1 , ..., xn } be a partition on [a, b] Let mk , Mk be inf and sup of f on Ik = [xk−1 , xk ]

Since f (x) = C, ∀ ∈ [a, b],


n
X n
X
∴ L(f, P ) = mk |Ik | = C |Ik | = C(b − a)
k=1 k=1

and
n
X n
X
U (f, P ) = Mk |Ik | = C |Ik | = C(b − a)
k=1 k=1
Therefore,
Z b Z b
f = inf U (f, P ) = sup L(f, P ) = f = C(b − a)
a P P a

∴ f is Riemann integrable on [a, b] and


Z b
Cdx = C(b − a)
a

1/x if 0 < x ≤ 1

Example 0.2.4 Define f : [0, 1] → R by f(x) =
0 if x = 0
Then
1
Z 1
dx
0 x
isn’t defined as a Riemann integral becuase f is unbounded. In fact, if

0 < x1 < x2 < ... < xn−1 < 1

is a partition of [0, 1], then

sup f = ∞
[0,x1 ]

so the upper Riemann sums of f are not well-defined.

Example 0.2.5 An integral with an unbounded interval of integration, such as


Z ∞
x dx
0
also isn’t defined as a Riemann integral. In this case, a partition of [1, ∞) into finitely many intervals contains
at least one unbounded interval, so the corresponding Riemann sum is not well-defined.

Example 0.2.6
0 0<x≤1

if
f (x) =
1 if x=0
is Riemann integrable, and
Z 1
f dx = 0.
0
To show this, let P = I1 , I2 , ..., In be a partition of [0, 1]. Then, since f (x) = 0 for x > 0,

Mk = sup f = 0, mk = inf f = 0 f or k = 2, 3, ...n


Ik Ik

The first interval in the partition is I1 = [0, x1 ], where 0 < x1 ≤ 1, and

M1 = 1, m1 = 0,

4
MA 1014-Mathematics Riemann Integration Dr Miuran Dencil

since f (0) = 1 and f (x) = 0 for 0 < x ≤ x1 . It follows that

U (f, P ) = x1 L(f, P ) = 0.
Thus, L(f ) = 0 and
U (f ) = inf{x1 : 0 < x1 ≤ 1} = 0,
so
U (f ) = L(f ) = 0
are equal, and the integral of f is 0.
We need to show that a particular function is integrable before actually integrating it.

0.2.1 The Cauchy criterion for integrability


The following theorem gives a criterion for integrability that is analogous to the Cauchy condition for the
convergence of a sequence.
Theorem 0.2.1 A bounded function f : [a, b] → R is Riemann integrable if and only if for every  > 0 there
exists a partition P of [a, b], which may depend on , such that
U (f, P ) − L(f, P ) ≤ 
Proof 0.2.3 (⇐) Let  > 0. If such a partition exists, then
U (f ) − L(f ) ≤ U (f, P ) − L(f, P ) < .
Since  is arbitrary, it follows that
U (f ) = L(f )
and so f is integrable.
(⇒) Assume f integrable on [a, b]. Then L(f ) = U (f ). So, given  > 0, there exist partitions P1 and P2 such
that
as Z b

L(f ) − < L(f, P1 ) ≤ f = L(f ) = U (f ) ≤ U (f, P2 ) ≤ U (f ) + /2
2 a
Let P = P1 ∪ P2 be the common refinement of P1 and P2 .
U (f, P ) − L(f, P ) ≤ U (f, P2 ) − L(f, P1 ) (0.8)
≤ (U (f ) + /2) − (L(f ) − /2) (0.9)
= /2 + /2 =  (0.10)
Thus, a partition P with the desired property exists.
Theorem 0.2.2 Every monotonic function f on [a, b] is integrable.
Proof 0.2.4 We assume f is increasing on [a, b] and leave the decreasing case as HW.

We also assume f (a) < f (b), since otherwise f would be a constant function. Since f (a) ≤ f (x) ≤ f (b) for all
x ∈ [a, b], f is clearly bounded on [a, b].

Let P = {x0 , x1 , ..., xn } be a partition of [a, b] satisfying ∆xk < δ = 


f (b)−f (a) for all k = 1, ..., n.

Then
n
X
U (f, P ) − L(f, P ) = [Mk − mk ](xk − xk−1 )
k=1
As f is monotonically increasing
n n
X X 
U (f, P ) − L(f, P ) = [f (xk ) − f (xk−1 )](xk − xk−1 ) ≤ [f (xk ) − f (xk−1 )]
f (b) − f (a)
k=1 k=1
As f (xn ) = f (b) and f (x0 ) = f (a), by telescoping series,

U (f, P ) − L(f, P ) ≤ [f (b) − f (a)] =
f (b) − f (a)
By Theorem [0.2.1], we have the result.

5
MA 1014-Mathematics Riemann Integration Dr Miuran Dencil

Theorem 0.2.3 If f is continuous on [a, b], then it is integrable.

Proof 0.2.5 Since f is continuous on the compact set [a, b], it is uniformly continuous on [a, b]. Let  > 0be
given. There exists δ > 0 such that

|f (x) − f (y)| <
2(b − a)
whenever x, y ∈ [a, b] and |x − y| < δ.

Let P = {x0 , x1 , ..., xn } be a partition of [a, b] satisfying ∆xk < δ for all k = 1, ..., n.

On any particular subinterval [xk−1 , xk ], by continuity, the function f achieves its minimum mk = f (yk ) and
its maximum Mk = f (zk ) at some yk , zk ∈ [xk−1 , xk ]. Since |yk − zk | ≤ ∆xk < δ, it follows that

Mk − mk = f (zk ) − f (yk ) < .
2(b − a)
Then
n n
X  X
U (f, P ) − L(f, P ) = (Mk − mk )∆xk < ∆xk = .
2(b − a)
k=1 k=1

By the Integrability Criterion, f is integrable on [a, b].

There exist functions which is integrable but not continuous.

Example 0.2.7 Define f : [0, 2] → R by

1 x 6= 1

if
f (x) =
0 if x=1

Let  satisfy 0 <  < 1. Then for the partition P = {0, 1 − 3 , 1 + 3 , 2} of [0, 2] we have,

 2 
U (f, P ) = 1.(1 − ) + 1. + 1.(1 − ) = 2
3 3 3

 2  2
L(f, P ) = 1.(1 − ) + 0. + 1.(1 − ) = 2 − .
3 3 3 3
Then U (f, P ) − L(f, P ) = 2 − (2 − 3 )
2
= 2
3 < . By the Integrability Criterion, f is integrable on [0, 2], and we
have Z 2
f = 2.
0

Note:

• If the set of points of discontinuity of a bounded function f : [a, b] → R is finite, then f is Riemann
integrable on [a, b].
• If the set of points of discontinuity of a bounded function f : [a, b] → R is finite number of limit points,
then f i integrable on [a, b].
i.e. A function may have infinitely many discontinuous points, but if the set of all discontinuous points
have finite number of limit points, then f is integrable on [a, b]

Theorem 0.2.4 f : [a, b] → R is bounded,and f is integrable on [c, b] for all c ∈ (a, b),then f is integrable on
[a, b]. An analogous result holds at the other endpoint.

6
MA 1014-Mathematics Riemann Integration Dr Miuran Dencil

Proof 0.2.6 Let  > 0. We will find a partition P such that

U (f, P ) − L(f, P ) < .

Now for any partition P , we may write


n
X
U (f, P ) − L(f, P ) = [Mk − mk ](xk − xk−1 )
k=1
n
X
= (M1 − m1 )(x1 − a) + [Mk − mk ](xk − xk−1 )
k=2

Since f is bounded on [a, b], there exists M > 0 such that |f (x)| < M for all x ∈ [a, b]. Choose x1 close enough
to a = x0 such that

(M1 − m1 )(x1 − a) < 2M (x1 − a) <
2
By hypothesis, f is integrable on [x1 , b], So, there exists a partition P1 of [x1 , b] such that

U (f, P1 ) − L(f, P1 ) < .
2
Now, let P = {a} ∪ P1 . Then
 
U (f, P1 ) − L(f, P1 ) = (M1 − m1 )(x1 − a) + (U (f, P1 ) − L(f, P1 )) < + .
2 2
By the Integrability Criterion, f is integrable on [a, b].

Theorem 0.2.5 Let f be a bounded function on [a, b] that is continuous on every subinterval [c, b] where c ∈
(a, b). Then f is integrable on [a, b]. An analogous result holds at the other endpoint.

Proof 0.2.7 Will prove this in next section.

Theorem 0.2.6 If f is a bounded function on [a, c] and integrable on [a, b] and [b, c], where a < b < c, then f
is integrable on [a, c].

Proof 0.2.8 Combining the previous Theorems (0.2.4) and (0.2.5) gives,

Example ( 0.2.8 Let f : [0, 1] → R such that


1 If x = n1 ; n ∈ N
f(x)=
0 otherwise
Z 1
Let’s show this is integrable function and find f (x)dx
0

We will use the Riemann Integral criterion to show that f is integrable on [0, 1]. Let  > 0 be given. We will
choose a partition P such that
U P, f ) − L(P, f ) < .
0
Since 1/n → 0, there exists N such that 1/n ∈ [0, ] for all n > N. So only finite number of n1 s lie in the in-
0
terval [, 1]. Cover these finite number of n1 s by the intervals [x1 , x2 ], [x3 , x4 ], ..., [xm−1 , xm ] such that xi ∈ [, 1]
for all i = 1, 2, .., m and the sum of the length of these m intervals is less than . Consider the partition
P = {0, , x1 , x2 , ..., xm }.

It is clear that U (P, f ) − L(P, f ) < 2 Hence by the Integral criterion the function is integrable. Since the lower
Z 1
integral is 0 and the function is integrable, f (x)dx = 0
0

0.3 Properties of the Integral


Notation: If a < b and f is integrable on [a, b], then
Z b Z a
f =− f
a b

7
MA 1014-Mathematics Riemann Integration Dr Miuran Dencil

Theorem 0.3.1 (Properties of Integrals) Assume f and g are integrable on [a, b].

1. f + g is integrable on [a, b] and


Z b Z b Z b
(f + g) = f+ g.
a a a

2. For k ∈ R, the function kf is integrable and


Z b Z b
kf = k f
a a
.
3. If m ≤ f (x) ≤ M on [a, b], then
Z b
m(b − a) ≤ f ≤ M (b − a)
a

4. If f (x) ≤ g(x)on [a, b],then


Z b Z b
f≤ g.
a a

5. The function |f | is integrable and Z


b Z b
f ≤ |f |.


a a

Proof 0.3.1 (1)


• Lets prove f + g is integrable on [a, b] and
Z b Z b Z b
(f + g) = f+ g.
a a a

Let  > 0. Then a partition P1 and P2 s.t. U (f, P1 ) − L(f, P1 ) < 


2 and U (f, P2 ) − L(f, P2 ) < 2 . Let P
be a common refinement of P1 and P2 . Then,

U (f, P ) − L(f, P ) < (0.11)
2

U (g, P ) − L(g, P ) < (0.12)
2
Therefore by equations (0.11), (0.12)

U (f + g, P ) − L(f + g, P ) ≤ U (f, P ) + U (g, P ) − L(f, P ) − L(g, P ) < .


∴ f+g is integrable.

Now, let Q be a partition on [a, b]. Then


Z b Z b Z b Z b
f+ g= f+ g ≤ U (f, Q) + U (g, Q) ≤ U (f + g, Q)
a a a a
Z b Z b Z b Z b
∴ f+ g≤ (f + g) = (f + g) also,
a a a a
Z b Z b Z b Z b
f+ g= f+ g ≥ L(f, Q) + L(g, Q) ≥ L(f + g, Q)
a a a a
Z b Z b Z b Z b
∴ f+ g≤ (f + g) = (f + g)
a a a a

Hence Z b Z b Z b
f+ g= (f + g)
a a a

8
MA 1014-Mathematics Riemann Integration Dr Miuran Dencil

• Let’s prove For k ∈ R, the function kf is integrable and


Z b Z b
kf = k f
a a

Proof 0.3.2 Since f is integrable for each  > 0 , P ∈ ℘[a, b], s.t

U (f, P ) − L(f, P ) <
k

Note that, by definition one can show that,

U (−f, P ) = −L(f, P ) and L(−f, P ) = −U (f, P )

Case I: k > 0

U (kf, P ) − L(kf, P ) = k(U (f, P ) − L(f, P )) < 


Hence, by Integrability Criterion, kf is integrable function.

n
X n
X
U (kf, P ) = M (kf, [xi−1 , xi ])(xi − xi−1 ) = kM (f, [xi−1 , xi ])(xi − xi−1 ) = kU (f, P )
i=1 i=1

As kf is integrable function ,
Z b Z b Z b
kf = kf = U (kf ) = inf{U (kf, P ); P ∈ ℘} = inf{kU (f, P ); P ∈ ℘} = k inf{U (f, P ); P ∈ ℘} = k f
a a a

Hence,
Z b Z b
kf = k f.
a a

Case II: k = −1,

U (−f, P ) − L(−f, P ) = −L(f, P ) + U (f, P )) = (U (f, P ) − L(f, P )) < 

Hence, by Integrability Criterion, −f is integrable function.

n
X n
X
U (−f, P ) = M (−f, [xi−1 , xi ])(xi − xi−1 ) = −M (f, [xi−1 , xi ])(xi − xi−1 ) = −U (f, P )
i=1 i=1

As −f is integrable function ,
Z b Z b Z b
−f = −f = U (−f ) = inf{U (−f, P ); P ∈ ℘} = inf{−U (f, P ); P ∈ ℘} = − inf{U (f, P ); P ∈ ℘} = − f
a a a

Case III: k < 0, k = −s; s > 0

Since f is R.I sf is R.I by Case I;

Since f is R.I −f is R.I by case II;


⇒ −sf is R.I
Z b Z b Z b Z b Z b
kf = −sf = − sf = −s f =k f
a a a a a

• Let’s prove, If m ≤ f (x) ≤ M on [a, b], then


Z b
m(b − a) ≤ f ≤ M (b − a)
a

9
MA 1014-Mathematics Riemann Integration Dr Miuran Dencil

Proof 0.3.3 For any given partition P = {x0 , x1 , ..., xn },

n
X Z b n
X
m(b − a) = mk (xk − xk−1 ) = L(f, P ) ≤ L(f ) ≤ f ≤ U (f ) ≤ U (f, P ) = Mk (xk − xk−1 ) ≤ M (b − a)
k=1 a k=1

Hence,
Z b
m(b − a) ≤ f ≤ M (b − a)
a

• The function |f | is integrable and Z


b Z b
f ≤ |f |.


a a

Proof 0.3.4 As −|f | ≤ f ≤ |f |, from the previous property


Z b Z b Z b
− |f | ≤ f≤ |f |.
a a a

Hence, Z
b Z b
f ≤ |f |.


a a

We now show |f | is integrable, For any subset S of [a, b], we have

M (|f |, S) − m(|f |, S) ≤ M (f, S) − m(f, S),

it follows that,
U (|f |, P ) − L(|f |, P ) ≤ U (f, P ) − L(f, P ), (0.13)

for all partitions P of [a, b]. As f is integrable,

U (f, P ) − L(f, P ) < 


Hence by equation [0.13]
U (|f |, P ) − L(|f |, P ) ≤ ,
by Integrability Criterion, |f | is integrable.

Theorem 0.3.2 If f and g are Riemann integrable, the f g is also Riemann integrable.

Proof 0.3.5 First we show that f 2 is integrable. Since f 2 = |f |2 without loss of generality (WLOG) let f (x) is
non-negative on [a, b]. Let  > 0 and |f (x)| bounded by k.

Since f ∈ R [where R- Riemann integrable functions], P = {x0 , x1 , ...xn } s.t


n
X 
U (f, P ) − L(f, P ) = (Mj − mj )|Ij | <
i=1
2k

Noe consider,
n n n
X X X 
U (f 2 , p) − L(f 2 , p) = (Mj2 − m2j )|Ij | = (Mj − mj )(Mj + mj )|Ij | ≤ 2k. (Mj − mj )|Ij | ≤ 2k =
i=1 i=1 i=1
2k

Hence f 2 is integrable on [a, b]


Now
(f + g)2 − f 2 − g 2
fg =
2
is integrable.

Theorem 0.3.3 If f and g are Riemann integrable, the M ax(f, g) and M in(f, g) is also Riemann integrable.

10
MA 1014-Mathematics Riemann Integration Dr Miuran Dencil

Proof 0.3.6 By definition,


|f − g| + f + g
M ax(f, g) :=
2
and
−|f − g| + f + g
M in(f, g) := .
2
Hence they are integrable.

Theorem 0.3.4 Assume f : [a, b] → R is bounded, and let c ∈ (a, b). Then f is integrable on [a, b] if and only
if it is integrable on [a, c] and [c, b]. In this case, we
Z b Z c Z b
f= f+ f.
a a c

Proof 0.3.7 ⇒) Assume f is integrable on [a, b]. Then for  > 0 there exists a partition P such that

U (f, P ) − L(f, P ) < .

Insert the point c into the partition P to get P 0 = {c} ∪ P . Since P 0 is a refinement of P ,

U (f, P 0 ) − L(f, P 0 ) < .

Let P1 = P ∩ [a, c] and P2 = P ∩ [c, b]. Then P1 is a partition of [a, c] and P2 is a partition of [c, b], and
U (f, P1 ) − L(f, P1 ) <  and U (f, P2 ) − L(f, P2 ) < , implying that f is integrable on both [a, c] and [c, b].

⇐)
Assume f is integrable on both [a, c] and [c, b]. Let  > 0. There exist partitions P1 and P2 of [a, c] and [c, b],
respectively, such that

U (f, P1 ) − L(f, P1 ) <
2
and

U (f, P2 ) − L(f, P2 ) < .
2
Then P = P1 ∪ P2 is a partition of [a, b] for which

U (f, P ) − L(f, P ) < .

and so f is integrable on [a, b]. Now, assuming that f is integrable on [a, b], [a, c], and [c, b], we’ll show that
Z b Z c Z b
f= f+ f.
a a c
Continuing with P = P1 ∪ P2 as earlier, where U (f, P ) − L(f, P ) < , we have
Z b Z c Z b
f ≤ L(f, P1 ) +  = L(f, P1 ) + L(f, P2 ) +  ≤ f+ f +
a a c
Since  > 0 is arbitrary,
Z b Z c Z b
f≤ f+ f.
a a c
Conversely,
Z c Z b Z c
f+ f ≤ U (f, P1 ) + U (f, P2 ) < L(f, P1 ) + L(f, P2 ) +  = L(f, P ) +  ≤ f + .
a c a

Because  > 0 is arbitrary,


Z b Z c Z b
f≥ f+ f.
a a c
So, we have
Z b Z c Z b
f= f+ f,
a a c
as desired.

11
MA 1014-Mathematics Riemann Integration Dr Miuran Dencil

We can also give a sequential characterization of integrability.

Theorem 0.3.5 A bounded function f : [a, b] → R is Riemann integrable if and only if there is a sequence (Pn )
of partitions such that
lim [U (f ; Pn ) − L(f ; Pn )] = 0.
n→∞

In that case,
Z b
f = lim U (f ; Pn ) = lim L(f ; Pn ).
a n→∞ n→∞

Proof 0.3.8 (⇐) First, suppose that the condition holds. Then, given  > 0, there is an n ∈ N such that
U (f ; Pn ) − L(f ; Pn ) < , so Theorem [0.2.1] implies that f is integrable and

U (f ) = L(f ).

Furthermore, since U (f ) ≤ U (f ; Pn ) and L(f ; Pn ) ≤ L(f ), we have

0 ≤ U (f ; Pn ) − U (f ) = U (f ; Pn ) − L(f ) ≤ U (f ; Pn ) − L(f ; Pn ).

Since the limit of the right-hand side is zero, the ‘squeeze’ theorem implies that
Z b
lim U (f ; Pn ) = U (f ) = f.
n→∞ a
It also follows that
Z b
lim L(f ; Pn ) = lim U (f ; Pn ) − lim (U (f ; Pn ) − L(f ; Pn )) = f.
n→∞ n→∞ n→∞ a

(⇒) Conversely, if f is integrable then, by 0.2.1 for every n ∈ N there exists a partition Pn such that
1
0 ≤ U (f ; Pn ) − L(f ; Pn ) < ,
n
and U (f ; Pn ) − L(f ; Pn ) → 0 as n → ∞.

Theorem 0.3.6 Let f be integrable over [a, b] and  > 0. Then δ > 0 s.t. ∀partitionP={x0 , x1 , ..., } with |P | < δ

Z b
X n
(ζ )I


f − f j j < 

a j=1

where ζj ∈ [xj−1 , xj ], j = 1, 2, ..., n


Z b Z b Z b Z b
Proof 0.3.9 Since f is integrable f = f = f. Choose δ > 0 s.t. U (f, p) < f +  and L(f, P ) >
a a a a
Z b
f −  ∀ partition P = {x0 , x1 , ..., } with |P | < δ. Let ζj ∈ [xj−1 − xj ], ∀j = 1, 2, ..., n.
a

Z b n
X Z b
f −  < L(f, P ) ≤ f (ζj )Ij ≤ U (f, P ) < f +
a j=1 a


Z b Xn
f (ζj )Ij <  ∀ partition P with |P | < δ.


f−
a j=1

0.4 Intermediate Value Theorem for Integrals.


Theorem 0.4.1 If f is a continuous function on [a, b], then for at least one x ∈ (a, b) we have

1 b
Z
f (x) = f
b−a a

12
MA 1014-Mathematics Riemann Integration Dr Miuran Dencil

Proof 0.4.1 Let M and m be the maximum and minimum values of f on [a,b]. If M = m, then f is a constant
1
Z b
function and f (x) = f all x ∈ [a, b].
b−a a
Otherwise, m < M then there exist distinct x0 and y0 in [a, b] satisfying f (x0 ) = m and f (y0 ) = M. Since each
Z b Z b Z b
function M − f and f − m is non-negative and not identically 0, then m< f< M. Thus
a a a
1 b
Z
m< f < M,
b−a a
Z b
and by the Intermediate Value Theorem for continuous functions , we have f (x) = f for some, x between
a
x0 and y0 . Since x ∈ [a, b], this completes the proof.

Theorem 0.4.2 (Generalization of Intermediate Value Theorem) f, g : [a, b] → R are continuous func-
tion and g(t) ≥ 0, ∀t ∈ [a, b]. Then ∃ at least one x ∈ [a, b] s.t
Z b Z b
f (t)g(t)dt = f (x) g(t)dt,
a a

that is Rb
f (t)g(t)dt
f (x) = a
Rb .
a
g(t)dt

Note: This is a generalization for I.V.T. By taking g(t) = 1 we may obtain the I.V.T for integrals.

Proof 0.4.2 Assume M and m are the maximum and minimum values of x ∈ [a, b].

mg(x) ≤ f (x)g(x) ≤ M g(x) as g(x)>0


Z b Z b Z b
m g(t)dt ≤ f (t)g(t)dt ≤ M g(t)dt.
a a a
Hence,
Rb
a
f (t)g(t)dt
m≤ Rb ≤M
a
g(t)dt
By I.V.T. for continuous functions this value must be taken by the function f on [a, b]. Therefore, ∃x ∈ [a, b] s.t
Rb
f (t)g(t)dt
f (x) = aR b .
a
g(t)dt

Example 0.4.1 Show that √


1
11 11 3
Z 2 p
≤ 1 − x2 dx ≤
24 0 36
1
Hint: Take f (x) = √ and g(x) = (1 − x2 )
1 − x2
Example 0.4.2 Show that
1 1
sin πx 2
Z
≤ dx ≤
π 0 1 + x2 π
An important question concerns when one can interchange limits and integrals. i.e. when
Z b Z b
lim fn (x)dx = f (x)dx
n→+∞ a a

true?. if the fn ’s are continuous and converge uniformly to f = lim fn on [a, b], then f is continuous and
n→∞
the statement holds. It turns out that if each fn is just Riemann integrable and fn → f uniformly, then f is
Riemann integrable and statement holds;

Theorem 0.4.3 Let fn be a sequence of R.I functions on [a, b] and suppose fn → f on [a, b]. Then f is R.I.
on [a, b] and
Z b Z b
lim fn (x)dx = f (x)dx.
n→+∞ a a

13
MA 1014-Mathematics Riemann Integration Dr Miuran Dencil

Proof 0.4.3 Let  > 0. Since fn → f uniformly on [a, b], there exists a number N such that |fn (x) − f (x)| <

2(b−a) for all x ∈ [a, b] and all n ≥ N . So in Particular, |fN (x) − f (x)| ≤ 2(b − a) dx ∀x ∈ [a, b].


As fN is Riemann integrable on [a, b] implies there’s a partition P = {a = t0 , t1 , t2 , ..., tn = b} s.t.

U (fN , P ) − L(fN , P ) < /2.

Now we consider the function f − fN with this partition.


n
X
U (f − fN , P ) − L(f − fN , P ) = [M (f − fN , [tk−1 , tk ]) − m((f − fN , [tk−1 , tk ])](tk − tk−1 )
k=1

Now as
 
− ≤ f (x) − fN (x) ≤
2(b − a) 2(b − a)
 
M (f − fN , [tk−1 , tk ]) ≤ and m(f − fN , [tk−1 , tk ]) ≥ − Therefore,
2(b − a) 2(b − a)
n
X
U (f − fN , P ) − L(f − fN , P ) = [M (f − fN , [tk−1 , tk ]) − m((f − fN , [tk−1 , tk ])](tk − tk−1 )
k=1
n   
X  
≤ (tk − tk−1 ) − −
2(b − a) 2(b − a)
k=1
n
X 
= (tk − tk−1 ) =
b−a
k=1

So we have for the partition P


U (f − fN , P ) − L(f − fN , P ) < 
Therefore, f − fN is R.I on [a, b]. Now f = (f − fN ) + fN . Hence f is R.I on [a, b]

Consequently n ≥ N implies

Z Z Z
b Z b b b Z b
 
fn (x)dx − f (x)dx = (fn (x) − f (x))dx ≤ |fn (x) − f (x)| dx ≤ dx =

a 2(b − a) 2

a a a a

Hence,
Z b Z b
lim fn (x)dx = f (x)dx.
n→+∞ a a


What happens if fn → f pointwise on [a, b]? One problem is that f need not be integrable even it is bounded
and each fn is integrable. In general it is not true statement. Nevertheless, there is an important theorem that
does apply to sequences of functions that converge pointwise.

Example 0.4.3
( Let {qk : k ∈ N} be an enumeration of the rationals in [0, 1] and define fn : [0, 1] → R by
1 If x=qk for k ≤ n,
fn (x) =
0 otherwise.
The each fn is Riemann integrable since it differs from the zero function at finitely many points. However,
fn → f pointwise on [0, 1] to the Dirichlet function f, which is not Riemann integrable.

Theorem 0.4.4 (Dominated Convergence Theorem) Suppose (fn ) is a sequence of integrable functions
on [a, b] and fn → f pointwise where f is an integrable function on [a, b]. If there exists an M > 0 such that
|fn (x)| ≤ M for all n and all x ∈ [a, b], then
Z b Z b
lim fn (x)dx = f (x)dx.
n→+∞ a a

Theorem 0.4.5 (Monotone Convergence Theorem) Suppose (fn ) is a sequence of integrable functions on
[a, b] such that f1 (x) ≤ f2 (x) ≤ ...for all x ∈ [a, b]. Suppose also that fn → f pointwise where f is an integrable
function on [a, b]. Then
Z b Z b
lim fn (x)dx = f (x)dx.
n→+∞ a a

14
MA 1014-Mathematics Riemann Integration Dr Miuran Dencil

This follows from the Dominated Convergence Theorem, because there exists an M > 0 such that |f1 (x)| ≤ M
and also |f (x)| ≤ M for all x ∈ [a, b]. This implies |fn (x)| ≤ M for all n and all x in [a, b], since

M ≤ f1 (x) ≤ fn (x) ≤ M

for all x.

Example 0.4.4 The function fn : [0, 1] → R defined by


n + cos x
fn (x) =
nex + sin x
converges uniformly on [0, 1] to f (x) = e−x , since for 0 ≤ x ≤ 1
n + cos x cos x − e−x sin x 1


nex + sin x − e −1
=
nex + sin x ≤ n .

1
n + cos x 1
1
Z Z
lim dx = e−1 dx = 1 − .
n→∞ 0 nex + sin x 0 e
Z π
sin(nx)
Example 0.4.5 Find lim dx
n→∞ π/2 nx

0.5 Fundamental Theorem of Calculus


There are two versions of the Fundamental Theorem of Calculus. Each says, roughly speaking, that differen-
tiation and integration are inverse operations. In fact, the first version says “the integral of the derivative of a
function is given by the function,” and the second version says “the derivative of the integral of a continuous
function is the function.”

Theorem 0.5.1 (Fundamental Theorem of Calculus- I) If g is a continuous function on [a, b] that is dif-
ferentiable on (a, b), and if g 0 is integrable on [a, b], then
Z b
g 0 = g(b) − g(a)
a

Proof 0.5.1 For any  > 0, there exists a partition P = {a = t0 , t1 , ..., tn = b} of [a, b] such that,

U (g, P ) − L(g, P ) < 

We apply the Mean Value Theorem to each interval [tk1 , tk ] to obtain xk ∈ (tk−1 , tk ) for which

(tk − tk−1 )g 0 (xk ) = g(tk ) − g(tk−1 ).


Hence we have
n
X n
X
g(b) − g(a) = g(tk ) − g(tk−1 ) = (tk − tk−1 )g 0 (xk )
k=1 k=1

It follows that
L(g 0 , P ) ≤ g(b) − g(a) ≤ U (g 0 , P );
By definition we know ,
Z b
L(g, P ) ≤ g 0 ≤ U (g 0 , P )
a
above inequalities imply
Z
b
g − (g(b) − g(a)) ≤ U (g 0 , P ) − L(g 0 , P ) < .
0


a
Since  is arbitary, the condition holds. i.e.
Z b
g 0 = g(b) − g(a)
a


15
MA 1014-Mathematics Riemann Integration Dr Miuran Dencil

xn+1
Example 0.5.1 If g(x) = , then g(x) = xn ,so
n+1
Z b
bn+1 an+1
xn = −
a n+1 n+1
Theorem 0.5.2 ( Integration by Parts.) If u and v are continuous functions on [a, b] that are differentiable
on (a, b), and if u0 and v 0 are integrable on [a, b], then
Z b Z b
u(x)v (x)dx +
0
u0 (x)v(x)dx = u(b)v(b) − u(a)v(a).
a a

Proof 0.5.2 Let g = uv; then g = uv + u v and g is integrable (as u, u0 , v, v 0 are integrable). Now by FTC
0 0 0

shows
Z b
g 0 = g(b) − g(a) = u(b)v(b) − u(a)v(a)
a
Z b
u(x)v 0 (x) + u0 (x)v(x) = u(b)v(b) − u(a)v(a)
a
Z b Z b
u(x)v (x) + 0
u0 (x)v(x) = u(b)v(b) − u(a)v(a).
a a


Theorem 0.5.3 (Fundamental Theorem of Calculus II) Let f be an integrable function on [a, b]. For x ∈
[a, b], let
Z x
F (x) = f (t)dt.
a
Then F is continuous on [a, b]. If f is continuous at x0 ∈ (a, b), then F is differentiable at x0 and
F 0 (x0 ) = f (x0 ).
Proof 0.5.3 Select M > 0 so that |f (x)| ≤ M for all x ∈ [a, b]. If x, y ∈ [a, b] and |xy| < δ = 
M where
x < y,say,then Z y Z y Z y
|F (y) − F (x)| = f (t)dt ≤ |f (t)|dt ≤ M dt = M (y − x) < .

x x x
This shows F is [uniformly] continuous on [a, b].
Now suppose f is continuous at x0 ∈ (a, b).Observe

F (x) − F (x0 ) 1 x
Z
= f (t)dt
x − x0 x − x0 x0
for x 6= x0 . Now, consider
F (x) − F (x0 ) 1 x
Z
− f (x0 ) = [f (t) − f (x0 )]dt.
x − x0 x − x0 x0

Let  > 0, since f is continuous at x0 , there exists δ > 0 such that


t ∈ (a, b) and |t − x0 | < δ imply |f (t) − f (x0 )| < ;
Hence
F (x) − F (x0 ) 1 1
Z x Z x
− f (x0 ) = [f (t) − f (x0 )]dt ≤ | |f (t) − f (x0 )|dt < 

|
x − x0 x − x0 x0 x − x0 x0
i.e.
F (x) − F (x0 )

− f (x0 ) < 


x − x0
for x ∈ (a, b) satisfying |x − x0 | < δ. We have just shown
F (x) − F (x0 )
lim = f (x0 ).
x→x0 x − x0
In other words,
F 0 (x0 ) = f (x0 ).

16
MA 1014-Mathematics Riemann Integration Dr Miuran Dencil

Theorem 0.5.4 If f is continuous on an open interval I containing the values of differentiable function y =
a(x) and y = b(x), then
Z b(x)
d
f (t)dt = f (b(x))b0 (x) − f (a(x))a0 (x).
dx a(x)

Proof 0.5.4 Z u
F (u) = f (t)dt.
t0

From FTC-II: F 0 (u) = f (u). Consider


"Z #
Z b(x) t0 Z b(x)
d d
f (t)dt = f (t)dt + f (t)dt
dx a(x) dx a(x) t0
"Z #
b(x) Z a(x)
d
= f (t)dt − f (t)dt
dx t0 t0

d
= [F (b(x)) − F (a(x))]
dx
= F 0 (b(x))b0 (x) − F 0 (a(x))a0 (x)
= f (b(x))b0 (x) − f (a(x))a0 (x)


Example 0.5.2 Solve followings
Z x2
d
• Find cos(t)dt
dx 0
Z x
• What is the function f defined by f (x) = f (t)dt + 2
0

• Let f be such that


Z x2
x sin(πx) = f (t)dt
0

Find f (1)

Theorem 0.5.5 (Change of Variable) Let u be a differentiable function on an open interval J such that u0
is continuous, and let I be an open interval such that u(x) ∈ I for all x ∈ J. If f is continuous on I, then f ◦ uis
continuous on J and Z b Z u(b)
f ◦ u(x)u0 (x)dx = f (u)du
a u(a)

17
MA 1014-Mathematics Riemann Integration Dr Miuran Dencil

0.6 Improper Riemann Integral


The Riemann integral is only defined for a bounded function on a compact interval (or a finite union of such
intervals). Nevertheless, we frequently want to integrate an unbounded function or a function on an infinite
interval. One way to interpret such an integral is as a limit of Riemann integrals; this limit is called an improper
Riemann integral.
Improper integrals: First, we define the improper integral of a function that fails to be integrable at one
endpoint of a bounded interval.

Definition 0.6.1 Suppose that f : (a, b] → R is integrable on [c, b] for every a < c < b. Then the improper
integral of f on [a, b] is
Z b Z b
f = lim+ f.
a →0 a+

The improper integral converges if this limit exists (as a finite real number), otherwise it diverges.

Similarly, if f : [a, b) → R is integrable on [a, c] for every a < c < b, then


Z b Z b−
f = lim+ f.
a →0 a

Example 0.6.1 If p > 0, the integral,


1
1
Z
dx
0 xp
isn’t defined as a Riemann integral since 1
xp is unbounded on (0, 1]. The corresponding improper integral is
1
1 1
1
Z Z
dx = lim+ dx.
0 xp →0  xp
For p 6= 1, we have
1
1 1 − 1−p
Z
dx = ,
 xp 1−p
so the improper integral converges if 0 < p < 1, with
1
1 1
1 1
Z Z
dx = lim+ dx = ,
0 xp →0  xp p−1
and diverges to ∞ if p > 1. Also, the integral also diverges (more slowly) to ∞ if p = 1 since
1
1 1
Z
dx = ln .
 xp 

We define improper integrals on an unbounded interval as limits of integrals on bounded intervals.

Definition 0.6.2 Suppose that f : [a, ∞) → R is integrable on [a, r] for every r > a. Then the improper integral
of f on [a, ∞) is
Z ∞ Z r
f = lim f.
a r→∞ a

Similarly, if f : (∞, b] → R is integrable on [r, b] for every r < b, then


Z b Z b
f = lim f.
∞ r→∞ −r

Example 0.6.2 Suppose p > 0. The improper integral,


Z ∞
1 1 r1−p − 1
Z r
dx = lim dx = lim
1 xp r→∞ 1 xp r→∞ 1 − p

converges to 1/(p − 1) if p > 1 and diverges to ∞ if 0 < p < 1.

It also diverges (more slowly) if p = 1 since

18
MA 1014-Mathematics Riemann Integration Dr Miuran Dencil


1 r
1
Z Z
dx = lim dx = lim ln r = ∞
1 x r→∞ 1 x r→∞

1
Thus, we get a convergent improper integral if the integrand xp decays sufficiently rapidly as x → ∞ (faster
than 1/x).

A divergent improper integral may diverge to ∞ (or ∞) as in the previous examples, or if the integrand changes
sign, it may oscillate.

More general improper integrals may be defined as finite sums of improper integrals of the previous forms. For
example, if f : [a, b]/{c} → R is integrable on closed intervals not including a < c < b, then
Z b Z c−δ Z b
f = lim+ f + lim+ f;
a δ→0 a →0 c+

and if f : R → R is integrable on every compact interval, then


Z ∞ Z c Z r
f = lim f + lim f,
−∞ s→∞ −s r→∞ c

where we split the integral at an arbitrary point c ∈ R. Note that each limit is required to exist separately.

Absolutely convergent improper integrals.


P
The convergence
P of improper integrals is analogousPto the convergence P of series. A series an an converges
absolutely if |an | converges, and conditionally if an converges but |an | diverges. We introduce a similar
definition for improper integrals and provide a test for the absolute convergence of an improper integral that is
analogous to the comparison test for series.
Z b Z b
Definition 0.6.3 An improper integral f is absolutely convergent if the improper integral |f | converges
Z b a Z b a

and conditionally convergent if f converges but |f | diverges.


a a
Z ∞ Z ∞
Theorem 0.6.1 If f dx is absolutely convergent, then the improper integral is convergent. i.e. |f |dx <
Z ∞ a a

∞ =⇒ f dx < ∞
a

Proof 0.6.1 By definition we know


= |f (x)| ≤ f (x) ≤ |f (x)|.
Now as,

f (x) ≤ |f (x)| (0.14)


f (x) + |f (x)| ≤ 2|f (x)| (0.15)
Z ∞ Z ∞
(f (x) + |f (x)|)dx ≤ 2 |f (x)|dx. (0.16)
a a
R∞ R∞
It’s given that a
|f (x)|dx < ∞. Therefore, by Comparison Theorem a
(f (x) + |f (x)|)dx < ∞.
Hence,
Z ∞ Z ∞ Z ∞
f (x)dx = (f (x) + |f (x)|)dx − f (x)dx < ∞.
a a a

Theorem 0.6.2 (Comparison Theorem) Suppose that f , g are two continuous functions for xa such that
0 ≤ g(x) ≤ f (x). Then
Z ∞ Z ∞
• f dx converges =⇒ gdx also converges
a a
Z ∞ Z ∞
• gdx diverges =⇒ f dx also diverges
a a

19
MA 1014-Mathematics Riemann Integration Dr Miuran Dencil

Proof 0.6.2 Let for all x ≥ a, 0 ≤ g(x) ≤ f (x); It follows that


Z t Z t
g(x)dx ≤ f (x)dx,
a a

Also, Z t Z t Z ∞
f (x)dx ≤ lim f (x)dx = f (x)dx.
a t→∞ a a
Therefore, Z t Z ∞
0 ≤ G(t) := g(x)dx ≤ f (x)dx
a a
R∞
Also G(t) is bounded above by a
f (x)dx. Therefore,
Z t Z ∞
lim g(x)dx = g(x)dx < ∞
t→∞ a a

On the other hand, Z ∞


g(x)dx
a
diverges. i.e. Z ∞
g(x)dx = ∞
a
Applying the definition of limits, For all M > 0, N s.t ∀t > N ,
Z t
g(x)dx > M
a

Since Z t Z t
g(x)dx ≤ f (x)dx,
a a
we have, Z t Z t
M< g(x)dx ≤ f (x)dx
a a
Therefore by definition, Z t
lim f (x)dx = ∞
t→∞ a
i.e Z ∞
f (x)dx = ∞
a

Example 0.6.3 Determine whether convergent or divergent.


1
Z 1
• √ dx
0 1 − x3
Z ∞
sin x
• dx
π x

f (x
Theorem 0.6.3 (Limit Comparison Theorem) Let f and g be continuous positive functions ∀x and lim =
x→∞ g(x)
k. Then
Z ∞ Z ∞
• 0 < k < ∞ =⇒ g dx converges ⇐⇒ f dx converges
a a
Z ∞ Z ∞
• k = 0 =⇒ g dx converges =⇒ f dx converges
a a
Z ∞ Z ∞
• k = ∞ =⇒ f dx converges =⇒ g dx converges
a a
R∞
Example 0.6.4 • 1
1
2+cos x+ln x dx

20
MA 1014-Mathematics Riemann Integration Dr Miuran Dencil

0.7 Beta and Gamma Functions


In this chapter we propose to discuss the Gamma and Beta functions. These functions arise in the solution of
physical problems and are also of great importance in various branches of mathematical analysis.

0.7.1 Euler’s integrals. Beta and Gamma functions


R1
Definition 0.7.1 (Beta function) The definite integral 0 xm−1 (1 − x)n−1 dx, for m > 0, n > 0 is known as
the Beta function and is denoted by B(m, n) [read as “Beta m, n”]. Beta function is also called the Eulerian
integral of the first kind.
Thus,
Z 1
B(m, n) = xm−1 (1 − x)n−1 dx, m > 0, n > 0.
0

Definition 0.7.2 (Gamma function ) The definite integral


Z ∞
e−x xn−i dx, n > 0,
0

is known as the Beta function and is denoted by Γ(n) [read as “Gamma n”]. Gamma function is also called the
Eulerain integral of the second kind.
Thus,
Z ∞
Γ(n) = e−x xn−1 dx, n > 0.
0
Remark: The First integral is valid only for m > 0 and n > 0 and the second integral is valid only for n > 0,
because it is for just these values of m and n that the above integrals are convergent.

0.7.2 Properties of Gamma function.


• Show that Γ(1) = 1.

Proof 0.7.1 By the definition of Gamma function,


Z ∞
e−x xn−1 dx, n > 0,
0
Z ∞ Z ∞
Γ(1) = e −x 1−1
x dx = e−x dx = 1
0 0

• Show that Γ(n + 1) = nΓ(n), n > 0.

Proof 0.7.2 We have from the definition of Gamma function,


Z ∞ Z ∞
Γ(n + 1) = −x n+1−1
e x dx = xn e−x dx
0 0
Z ∞
= [x (−e )]0 −
n −x ∞
(nxn−1 )(−e−x )dx
0
Z ∞
xn
= − lim x + 0 + n (xn−1 )(e−x )dx
x→∞ e 0
Z ∞
=n (x n−1
)(e )dx = nΓn
−x
0

• If n is a non-negative integer, then Γ(n + 1) = n!

Proof 0.7.3 We known that for n > 0, we have (from property II)
Γ(n + 1) = nΓ(n) = nΓ(n − 1 + 1) = n(n − 1)Γ(n − 1)
= n(n − 1)(n − 2)Γ(n − 2),
= n(n − 1)(n − 2). . . 3.2.1Γ(1),

21
MA 1014-Mathematics Riemann Integration Dr Miuran Dencil

0.7.3 Extension of definition of Gamma function Γ(n) for n < 0.


When n > 0, we known that Γ(n + 1) = nΓ(n) so that

Γ(n + 1)
Γ(n) = .
n
Let −1 < n < 0. Then −1 < n =⇒ n + 1 > 0 so that Γ(n + 1) is well defined by definition. Thus Γ(n) is
defined for −1 < n < 0 Similarly, Γ(n) is given by

Γ(n + 1)
Γ(n) = ,
n
for −2 < n < −1, −3 < n < −2 and so on.Thus Γ(n) defines for all values of n except n = 0, −1, −2, −3, ...

Lemma 0.7.1 For every S > 0 the improper integral


Z ∞
e−sx dx < ∞
0

Lemma 0.7.2 Let n be a positive integer. Then

tn−1
lim =0
x→∞ et/2
Z ∞
Theorem 0.7.1 For every n > 0 the improper integral Γ(n) = e−x xn−1 dx is convergent.
0

Proof 0.7.4 We will prove this in three cases: Case I: n is a positive integer ≥ 1 By 0.7.2;

xn−1
lim = 0.
x→∞ ex/2

By definition of limits, with  = 1 , ∃ M s.t. ∀x ≥ M


n−1
x
ex/2 < 1

That means,
xn−1 < ex/2
And
e−x xn−1 < e−x ex/2 = e−x/2
By comparison integral theorem,
Z ∞ Z ∞
−x n−1
e x dx < e−x/2 dx < ∞ , by 0.7.1
M M


Z ∞
e−x xn−1 dx < ∞.
M

Hence Γ(n) is convergent for all positive integer.

Case II: Let y = n ≥ 1 be any real


√ number. Let [y] be the largest integer, so that [y] < y < [y] + 1
For example :[π] = 3, [e] = 2, [ 2] = 1
For x > 0,
0 ≤ e−x xy−1 ≤ e−x x[y] .
Since Z ∞
e−x x[y] = Γ([y] + 1)
0

is convergent [by Case I], we have Z ∞


e−x x[y] < ∞,
0

22
MA 1014-Mathematics Riemann Integration Dr Miuran Dencil

by comparison integral theorem.

Case III: Now let 0 < n < 1 Note that n < 1 =⇒ n < 2 =⇒ n − 1 < 1 =⇒ xn−1 < x
1 xn−1 x
≤ < x/2
ex/2 e x/2 e
By applying Sandwich Theorem
1 xn−1 x
lim ≤ lim < lim x/2
x→∞ ex/2 x→∞ ex/2 x→∞ e

Hence
xn−1
=0 lim
x→∞ ex/2

By definition of limits, with  = 1 , ∃ M s.t. ∀x ≥ N


n−1
x
ex/2 < 1 =⇒ x < et/2 =⇒ e−t xn−1 < e−t/2
n−1

By comparison integral theorem, Z ∞


e−x xn−1 dx < ∞
N
Z N
Now consider the integral e−x xn−1 dx. Note that when 0 < n < 1,
0

lim e−x xn−1 = ∞.


x→0

Z N
∴ The function e −x n−1
x is unbounded at x = 0 or in the interval [0, N ]. Therefore, e−x xn−1 dx. is an
0
improper integral of Type I.

Note that for x > 0,


e−x xn−1 ≤ xn−1
Then,
Z N Z N
e −x n−1
x dx = lim+ e−x xn−1 dx
0 →0 
Z N
≤ lim+ xn−1 dx
→0 
n
x N
= lim+ |
n 
→0
N n − n
= lim+
→0 n
Nn
= < ∞.
n
Z N
e−x xn−1 dx < ∞
0
.

Thus, for all n > 0, Z ∞


Γ(n) = e−x xn−1 dx < ∞.
0

23
MA 1014-Mathematics Riemann Integration Dr Miuran Dencil

0.7.4 Transformation of Gamma function.


• Form I: Show that ∞
1
Z
1/n
Γ(n) = e−x , n > 0.
n 0

Proof 0.7.5 By definition, Z ∞


Γ(n) = e−x xn−1 dx, n > 0.
0

Put xn = t so that nxn−1 dx = dt. Then by definition,


1 ∞ −t1/n
Z
Γ(n) = e dt, n > 0.
n 0
Hence the proof.

Let n = 1/2, Then Z ∞ √


2
Γ(1/2) = 2 e−x dx = π
0

• Form II. Show that ∞


Γ(n)
Z
e−kx xn−1 dx = ; n > 0, k > 0.
0 kn

Proof 0.7.6 By definition, Z ∞


Γ(n) = e−x xn−1 dx, n > 0.
0
Put x = kt so that dx = kdt, Then this equation gives
Z ∞
Γ(n) = e−kx k n−1 tn−1 kdx
0

or in other words,

Γ(n)
Z
e−kx xn−1 dx =
0 kn

• Form III: Show that n−1


1
1
Z 
Γ(n) = log dx; n > 0.
0 x

Proof 0.7.7 By definition, Z ∞


Γ(n) = e−x xn−1 dx, n > 0.
0

P ute˘x = t so that ˘e˘x dx = dt, then x = log 1t . Then this equation gives,
n−1 n−1
0
1 1
Z  Z 1
Γ(n) = − log dx = log dx
1 t 0 t

• Form IV:Show that Z ∞


2
Γ(n) = 2 e−x x2n−1 dx, n > 0.
0

Proof 0.7.8 By definition, Z ∞


Γ(n) = e−x xn−1 dx, n > 0.
0

Put x = t2 so that dx = 2tdt. Then this equation reduced to


Z x
2
Γ(n) = e−t (t2 )n−1 2tdt,
0

In otherwords, Z ∞
2
Γ(n) = 2 e−x x2n−1 dx, n > 0.
0

24
MA 1014-Mathematics Riemann Integration Dr Miuran Dencil

Example 0.7.1 Solve following problems:


a) Evaluate Z ∞
x4 e−x dx
0

b) Evaluate Z ∞
x6 e−2x dx
0

c) Compute Γ(− 21 ),
d) If nis a positive integer and m > ˘1,prove that
1
(−1)n n!
Z
xm (log x)n dx =
0 (m + 1)n+1

e) Evaluate
Z 1
dx
− log(x)
p
0

25

You might also like